Login

Welcome, Guest. Please login or register.

March 29, 2024, 10:40:01 am

Author Topic: Harder Physics Questions  (Read 20992 times)  Share 

0 Members and 2 Guests are viewing this topic.

full of electrons

  • Victorian
  • Fresh Poster
  • *
  • Posts: 3
  • Respect: +1
Re: Harder Physics Questions
« Reply #60 on: February 04, 2010, 12:39:01 am »
0
Hi everyone, sorry but I've just joined this conversation. could we go back a bit to the gravity problem?

I'm going to ignore the need for the buoyancy force. Using gauss' law (as QuantumJG did)....
I get a value of g = 5.91*10^4 m/s^2
Orbital velocity of earth: 9.4*10^7 m/s

Orbital period: 1.00*10^4 seconds.

/0

  • Victorian
  • ATAR Notes Legend
  • *******
  • Posts: 4124
  • Respect: +45
Re: Harder Physics Questions
« Reply #61 on: February 04, 2010, 01:44:53 am »
0
OK.

A solenoid with a current i and n loops is placed on a ramp inclined at an angle theta to the horizontal. Consider the magnetic field of the earth and calculate the acceleration of the solenoid down the slope in respect to position.

(You'll need to introduce a few basic constants with the algebra)

Edit: To make life easier and to simplify the maths, presume that the solenoid can immediately roll without slipping.

Been a while since I've done (few) umep problems, hope it's close

If we assume the magnetic field is perpendicular to the direction of the slope. The current will not affect the acceleration due to the right hand slap rule cancelling out over the circular loops. The problem is then reduced to a rolling tube.

The same thing happens if the magnetic field is parallel to the slope.

Take the contact point between the solenoid and the ramp to be the pivot.

By the parallel-axis theorem









(I know this solution probably isn't what you had in mind and I might have misunderstood but oh well...

In the meantime, have a go at this problem. I hope it isn't pushing the math limits, but it is a variation on a classic electromagnetism problem.

We have a line of positive charge of length L and total charge Q. At one end, a certain distance 'z' (or call it whatever you like)
 above the line of charge is a point P. Find the Electric field at that point. Express it in terms of vectors in the x and y directions.)

[There are several ways to get the answer]
« Last Edit: February 04, 2010, 02:07:42 am by /0 »

appianway

  • Guest
Re: Harder Physics Questions
« Reply #62 on: February 04, 2010, 08:26:10 pm »
0
Oops, I think I wrote solenoid when I meant wires wrapped around a cardboard tube (so not like a coil like a solenoid, wrapped lengthwise around...). So yes, that changes it significantly...

And I think I've done that question before. I'll try and find my solution or just do it again :)

Cthulhu

  • Guest
Re: Harder Physics Questions
« Reply #63 on: February 04, 2010, 10:48:40 pm »
0
I've done a question like that. I used logic to answer it though and the rod was semi-infinite and I ended up getting it wrong. so......... *facepalm*

/0

  • Victorian
  • ATAR Notes Legend
  • *******
  • Posts: 4124
  • Respect: +45
Re: Harder Physics Questions
« Reply #64 on: February 12, 2010, 10:49:39 pm »
0
Oops, I think I wrote solenoid when I meant wires wrapped around a cardboard tube (so not like a coil like a solenoid, wrapped lengthwise around...). So yes, that changes it significantly...

And I think I've done that question before. I'll try and find my solution or just do it again :)

Sorry I still don't think I really understand the problem, but it definitely sounds like an interesting one, and it would be great if you could post a diagram and/or solution to it.

If anyone is interested in the solution to my previous problem, it's




Anyway,

POST MORE PROBLEMS!

Edmund

  • Dr. Ruler Snapper
  • Victorian
  • Part of the furniture
  • *****
  • Posts: 1849
  • Respect: +95
Re: Harder Physics Questions
« Reply #65 on: February 17, 2010, 09:27:58 pm »
0
Yeah any new problems to share?  :-\
2007-2008 VCE ATAR 90.15
2009-2011 BSc (Unimelb)
2012-2015 DDS (Unimelb)

Booksale: Drugs That Shape Society, Forests in a Global Context

/0

  • Victorian
  • ATAR Notes Legend
  • *******
  • Posts: 4124
  • Respect: +45
Re: Harder Physics Questions
« Reply #66 on: February 18, 2010, 01:03:01 pm »
0
The pressure underwater at a depth is ,
where is atmospheric pressure,
and is density.

A circular pool has radius of 10m, and the water depth is 1.5m. Find the total force exerted on the walls.

Extension: Prove the formula (i.e. that pressure depends on height)


[Let's try to get this into a Physics Marathon, so if you solve a question you have to post another one!]
« Last Edit: February 18, 2010, 05:29:06 pm by /0 »

Cthulhu

  • Guest
Re: Harder Physics Questions
« Reply #67 on: February 18, 2010, 05:00:11 pm »
0
Now I'm no physicist and I haven't done fluids in a long time but intuition and total guessing tells me that the area/volume of the pool doesn't come into play here its just





or maybe it's just

*shrug*


/0

  • Victorian
  • ATAR Notes Legend
  • *******
  • Posts: 4124
  • Respect: +45
Re: Harder Physics Questions
« Reply #68 on: February 18, 2010, 05:29:59 pm »
0
Ugh, my bad, it's meant to read total 'Force' exerted on the walls. (Of course, since Pressure is per unit area)

Cthulhu

  • Guest
Re: Harder Physics Questions
« Reply #69 on: February 18, 2010, 05:38:22 pm »
0
ohhhhhhhhhhhhhhhhhhhhhhhhhhhhhhhhhhhhh I sort of know what to do then
*goes back to work*

Cthulhu

  • Guest
Re: Harder Physics Questions
« Reply #70 on: February 18, 2010, 07:38:10 pm »
0
Again I'm only speculating the answer/working/method




From above

and
So

Or as before using -14700Pa

Again. Only speculation I have no idea what I'm doing/where I'm going with this. I suck at fluids :-[
« Last Edit: February 18, 2010, 07:40:46 pm by Cthulhu »

/0

  • Victorian
  • ATAR Notes Legend
  • *******
  • Posts: 4124
  • Respect: +45
Re: Harder Physics Questions
« Reply #71 on: February 19, 2010, 12:30:49 am »
0
The key thing here is that P is not constant, it increases with depth. Also, the area we're seeking is the surrounding walls, not the bottom. But apart from that you've got mostly the right idea.

Try graphing P against h and see if you can draw any conclusions :P

Hint: average!

enwiabe

  • Putin
  • ATAR Notes Legend
  • *******
  • Posts: 4358
  • Respect: +529
Re: Harder Physics Questions
« Reply #72 on: February 19, 2010, 02:16:23 am »
0
you have to use a pressure trapezoid

and multiply that by the surface area of the cylinder which will be 2pi*r*h
« Last Edit: February 19, 2010, 02:18:05 am by enwiabe »

QuantumJG

  • Victorian
  • Part of the furniture
  • *****
  • Posts: 1748
  • Applied Mathematics Student at UoM
  • Respect: +82
Re: Harder Physics Questions
« Reply #73 on: February 19, 2010, 10:39:12 am »
0
The pressure underwater at a depth is ,
where is atmospheric pressure,
and is density.

A circular pool has radius of 10m, and the water depth is 1.5m. Find the total force exerted on the walls.

Extension: Prove the formula (i.e. that pressure depends on height)


[Let's try to get this into a Physics Marathon, so if you solve a question you have to post another one!]

Cool, let's do some calculus;

F = p x A

dF = p x dA

dF = (Po + ρgh) x 2πr x dh

F = (Po + ρgh) x 2πr x dh
 
  = 2πrPo dh + 2πrρg hdh

  = 2πrPo h|01.5 + πrρgh2|01.5

F = 1.106x107N

I'll try the challenge question now!
2008: Finished VCE

2009 - 2011: Bachelor of Science (Mathematical Physics)

2012 - 2014: Master of Science (Applied Mathematics/Mathematical Physics)

2016 - 2018: Master of Engineering (Civil)

Semester 1:[/b] Engineering Mechanics, Fluid Mechanics, Engineering Risk Analysis, Sustainable Infrastructure Engineering

Semester 2:[/b] Earth Processes for Engineering, Engineering Materials, Structural Theory and Design, Systems Modelling and Design

/0

  • Victorian
  • ATAR Notes Legend
  • *******
  • Posts: 4124
  • Respect: +45
Re: Harder Physics Questions
« Reply #74 on: February 19, 2010, 11:23:13 am »
0
Correct QuantumJG! (at least I think it is)

There is another solution that doesn't involve calculus.

The pressure at the bottom is , and the pressure at the top is just

Since is a linear function of , the average pressure is .

Thus we have